LSAT and Law School Admissions Forum

Get expert LSAT preparation and law school admissions advice from PowerScore Test Preparation.

User avatar
 Dave Killoran
PowerScore Staff
  • PowerScore Staff
  • Posts: 5852
  • Joined: Mar 25, 2011
|
#88231
Complete Question Explanation
(The complete setup for this game can be found here: lsat/viewtopic.php?f=174&p=88228#p88228)

The correct answer choice is (B).

In a game with templates, in Global questions the best approach is to scan the two templates in response to each answer choice.

Answer choice (A) is incorrect because, as shown in Template #2, Z, Y, and W can all sign with the same agency.

Answer choice (B) is the correct answer. In each template, F signs only one or two performers. Interestingly, students who eliminate answer choice (E) in question #2 typically find this answer choice easy to select.

Answer choice (C) is incorrect because Template #1, for instance, easily shows how F can sign two performers whereas S signs only 1 performer (or vice versa).

Answer choice (D) is incorrect because as explained in the discussion of the fourth rule, T and W do not form a block and they can sign with different agencies.

Answer choice (E) is incorrect because as shown in Template #2, W can sign with F.
 deck1134
  • Posts: 160
  • Joined: Jun 11, 2018
|
#47645
Hi all,

I am really struggling with question 3. I cannot see how B is the answer. I originally chose A.

I set up the templates with T, X, and W on F. Why is this incorrect?
 Adam Tyson
PowerScore Staff
  • PowerScore Staff
  • Posts: 5153
  • Joined: Apr 14, 2011
|
#47650
Consider the effect of ALL the rules, deck, including that Z and Y sign with the same agency as each other and, from the scenario, that each agency signs at least one performer, and you'll discover that your hypothetical will violate one of those requirements.

Get the most out of your LSAT Prep Plus subscription.

Analyze and track your performance with our Testing and Analytics Package.